Rekursive -> explizite Formel

Neue Frage »

Paula1982 Auf diesen Beitrag antworten »
Rekursive -> explizite Formel
Hallöchen!
Hab da ein kleines Problem:
Ich soll aus der Formel:
und dem Glied
die explizite Formel erstellen.
Leider hänge ich irgendwo fest oder habe mich total vertan...
Habe erstmal vermutet, dass es eine Formel 2. Grades ist, da wir das auch in der Vorlesung hatten, nicht geklappt. 3. Grades auch nicht geklappt....
Kann ich irgendwie erkennen welchen Grad ich brauche?
Habe die ersten fünf Glieder bestimmt und jedes Mal nen Gleichungsystem aufgestellt.
Ich denke ich habe mich verrechnet aber habe inzwischen schon 100mal nachgerechnet und finde den Fehler nicht...
Wäre nett, wenn mir jeman nen Ansatz verraten könnte.

Lieben Gruß
Paula
IfindU Auf diesen Beitrag antworten »
RE: Rekursive -> explizite Formel
Manchmal ist weniger Grad mehr. Rechne doch ein paar Folgenglieder aus und schau ob sie dir bekannt vorkommt.
wisili Auf diesen Beitrag antworten »
RE: Rekursive -> explizite Formel
-- zu spät
Paula1982 Auf diesen Beitrag antworten »

Habe die ersten sechs berechnet, stehe aber voll auf dem Schlauch...
Ist das mit dem Gleichungssystem nun der falsche Weg???
IfindU Auf diesen Beitrag antworten »

Zeig uns doch die ersten 6.
Paula1982 Auf diesen Beitrag antworten »

a1=3*1+1=4
a2=3*4+1=13
a3=3*13+1=40
a4=3*40+1=121
a5=3*121+1=366
a6=3*366+1=730
 
 
IfindU Auf diesen Beitrag antworten »
RE: Rekursive -> explizite Formel
Zitat:
und dem Glied


Kann es sein, dass es ist? - was die Sache nicht mehr so trivial macht.
Paula1982 Auf diesen Beitrag antworten »

Oh ja du hast recht, habe die 3 vergessen. SORRY!!!
IfindU Auf diesen Beitrag antworten »

Wir können die Gleichung ja mal rückwärts angehen:


Jetzt kannst du dir ja überlegen wie das aussieht wenn man das n-mal macht. Und da das nicht wirklich aussieht wie ein Poylnom wirst du bei deinem Gradansatz Probleme bekommen.
Paula1982 Auf diesen Beitrag antworten »

Spontan auffällig:


also denke ich mal eher an exponentielles Wachtum.... Oder?
IfindU Auf diesen Beitrag antworten »

Von der Form solltest du wenigstens eine Vermutung haben wie das ganze immer weitergeht und das harmlos in eine Summenformel packen. Die Vermutung kannst du dann durch einsetzen überprüfen und die geometrische Reihe sollte dir beim Rest helfen.
Paula1982 Auf diesen Beitrag antworten »

Ohje, Summenformel, hatte ich ja schon ewig nicht. Kann mir auch nicht vorstellen, dass das hier gefordert sein soll, da wir sogar Leute aus dem zweiten Semester in der Veranstaltung haben....
NAja dann schaue ich mal, ob ich weiter komme und melde mich später bestimmt nochmal wieder...
DANKE!!!
wisili Auf diesen Beitrag antworten »

Probier mal a_n = p*3^n + q.

(Setze n=0 und dann noch n=1 ein; bestimme dann p und q.)
Paula1982 Auf diesen Beitrag antworten »

Der Tipp mit der geometrischen Reihe war klasse smile
DANKE!!!!!!
Müsste die Formel haben!!!



War nur nochmal nach der Formel für die geometrische Reihe schauen und dann darin meine 3 einsetzen.
Beweisen kann ich doch ganz einfach mit vollständiger Induktion....
Nochmal dickes Danke für den Tipp!!! Mit Zunge
IfindU Auf diesen Beitrag antworten »

Du hast dich irgendwo vertan, da a_0 = 0 wäre und nicht 1, ist aber wohl nur eine Verschiebung.
Paula1982 Auf diesen Beitrag antworten »

Für mich gehört die 0 nie zu den natürlichen Zahlen smile
Mein Dozent nimmt sie aber immer mit...
Muss also nochmal kurz nachrechnen.
IfindU Auf diesen Beitrag antworten »



Das musst du nur so hinschreiben und schon passt es. Natürlich kann man das noch schöner schreiben: 1-3^n ist nicht so schön, da 3^n immer größergleich 1 ist und der Zähler automatisch negativ wird und der Nenner genauso negativ ist, und auch zusammengefasst werden kann.

Ansonsten bist du fertig. Freude
Paula1982 Auf diesen Beitrag antworten »

Danke, dann werd ich mich jetzt mal an die vollständige Induktion machen!
Neue Frage »
Antworten »



Verwandte Themen

Die Beliebtesten »
Die Größten »
Die Neuesten »